Solving $f(yf(x)+x/y)=xyf(x^2+y^2)$ over the reals The 2019 Stack Overflow Developer Survey...

Why is Grand Jury testimony secret?

Is three citations per paragraph excessive for undergraduate research paper?

Are there any other methods to apply to solving simultaneous equations?

How to reverse every other sublist of a list?

Does duplicating a spell with Wish count as casting that spell?

"To split hairs" vs "To be pedantic"

It's possible to achieve negative score?

What is the motivation for a law requiring 2 parties to consent for recording a conversation

What does "sndry explns" mean in one of the Hitchhiker's guide books?

How are circuits which use complex ICs normally simulated?

What could be the right powersource for 15 seconds lifespan disposable giant chainsaw?

Why did Howard Stark use all the Vibranium they had on a prototype shield?

What are my rights when I have a Sparpreis ticket but can't board an overcrowded train?

Why don't Unix/Linux systems traverse through directories until they find the required version of a linked library?

Where to refill my bottle in India?

Where does the "burst of radiance" from Holy Weapon originate?

What is the steepest angle that a canal can be traversable without locks?

Is flight data recorder erased after every flight?

Pristine Bit Checking

Limit the amount of RAM Mathematica may access?

What can other administrators access on my machine?

How to create dashed lines/arrows in Illustrator

On the insanity of kings as an argument against monarchy

How to change the limits of integration



Solving $f(yf(x)+x/y)=xyf(x^2+y^2)$ over the reals



The 2019 Stack Overflow Developer Survey Results Are InSolving for the implicit function $fleft(f(x)y+frac{x}{y}right)=xyfleft(x^2+y^2right)$ and $f(1)=1$Solving for the implicit function $fleft(f(x)y+frac{x}{y}right)=xyfleft(x^2+y^2right)$ and $f(1)=1$Show that $frac{xy}{z} + frac{xz}{y} + frac{yz}{x} geq x+y+z $ by considering homogeneityGiven prime numbers $p,q$ and $r$, $p|qr − 1$, $q|rp − 1$, and $r|pq − 1$. What is the value of all possible $pqr$?Determining information in minimum trials (combinatorics problem)Determine all functions satisfying $fleft ( f(x)^{2}y right )=x^{3}f(xy)$Prove $sumlimits_text{cyc}frac{a}{a+(n-1)b}geq 1$Solving the functional equation $f(xf(x)+yf(y))=xy$ over positive realsProve that $(k^3)!$ is divisible by $(k!)^{k^2 + k + 1}$. Proof using properties of greatest integer function?Strange Examples of Involutory Functions?Funcional equation $f(xyf(x+y))=f(x)+f(y)$












19












$begingroup$



Find all functions $f:mathbb{R}to mathbb{R}$ such that $f(1)=1$ and for all real numbers $x$ and $y$ with $y neq 0$, $$fBigg (yf(x)+frac{x}{y}Bigg)=xyf(x^2+y^2)$$




This seems quite hard. $f(x)=begin{cases}frac{1}{x}, xneq 0 \ 0, x=0 end{cases}$ works by inspection, as does $f(x)=0$ (though I'm not sure if this is legitimate. If we set $y=1$, we have $f(f(x)+x)=xf(x^2+1)$. If we set $x=1$, we have $f(frac{y^2+1}{y})=yf(y^2+1)$. which seems to imply $f(f(x)+x)=f(frac{x^2+1}{x})$. If I could show that $f$ is injective I could go further, but I'm stuck - subbing in other values doesn't really seem to lead anywhere either.



I believe this problem came from an Olympiad camp.










share|cite|improve this question











$endgroup$












  • $begingroup$
    Are you sure it is $yneq 1$ and not $yneq 0$?
    $endgroup$
    – Redundant Aunt
    Mar 21 at 11:55










  • $begingroup$
    @RedundantAunt should be $yneq 0$ - fixed, thanks!
    $endgroup$
    – user574848
    Mar 21 at 11:59










  • $begingroup$
    The problem seems really hard to me! Are you sure that it has a nice solution or might there exist pathological solutions to this functional equation?
    $endgroup$
    – Redundant Aunt
    Mar 26 at 8:12










  • $begingroup$
    @RedundantAunt this question was shared to me - unfortunately, I can’t solve it either! (Hence why I’m asking)
    $endgroup$
    – user574848
    Mar 26 at 11:35








  • 3




    $begingroup$
    This is a duplicate of Solving for the implicit function $fleft(f(x)y+frac{x}{y}right)=xyfleft(x^2+y^2right)$ and $f(1)=1$, and AOPS links artofproblemsolving.com/community/c6h411400p2923354 , artofproblemsolving.com/community/c6h148711 and artofproblemsolving.com/community/c6h358223. And the original source maa.org/sites/default/files/pdf/AMC/a-activities/a6-mosp/… (Problem 7.4)
    $endgroup$
    – Sil
    Mar 31 at 11:10
















19












$begingroup$



Find all functions $f:mathbb{R}to mathbb{R}$ such that $f(1)=1$ and for all real numbers $x$ and $y$ with $y neq 0$, $$fBigg (yf(x)+frac{x}{y}Bigg)=xyf(x^2+y^2)$$




This seems quite hard. $f(x)=begin{cases}frac{1}{x}, xneq 0 \ 0, x=0 end{cases}$ works by inspection, as does $f(x)=0$ (though I'm not sure if this is legitimate. If we set $y=1$, we have $f(f(x)+x)=xf(x^2+1)$. If we set $x=1$, we have $f(frac{y^2+1}{y})=yf(y^2+1)$. which seems to imply $f(f(x)+x)=f(frac{x^2+1}{x})$. If I could show that $f$ is injective I could go further, but I'm stuck - subbing in other values doesn't really seem to lead anywhere either.



I believe this problem came from an Olympiad camp.










share|cite|improve this question











$endgroup$












  • $begingroup$
    Are you sure it is $yneq 1$ and not $yneq 0$?
    $endgroup$
    – Redundant Aunt
    Mar 21 at 11:55










  • $begingroup$
    @RedundantAunt should be $yneq 0$ - fixed, thanks!
    $endgroup$
    – user574848
    Mar 21 at 11:59










  • $begingroup$
    The problem seems really hard to me! Are you sure that it has a nice solution or might there exist pathological solutions to this functional equation?
    $endgroup$
    – Redundant Aunt
    Mar 26 at 8:12










  • $begingroup$
    @RedundantAunt this question was shared to me - unfortunately, I can’t solve it either! (Hence why I’m asking)
    $endgroup$
    – user574848
    Mar 26 at 11:35








  • 3




    $begingroup$
    This is a duplicate of Solving for the implicit function $fleft(f(x)y+frac{x}{y}right)=xyfleft(x^2+y^2right)$ and $f(1)=1$, and AOPS links artofproblemsolving.com/community/c6h411400p2923354 , artofproblemsolving.com/community/c6h148711 and artofproblemsolving.com/community/c6h358223. And the original source maa.org/sites/default/files/pdf/AMC/a-activities/a6-mosp/… (Problem 7.4)
    $endgroup$
    – Sil
    Mar 31 at 11:10














19












19








19


4



$begingroup$



Find all functions $f:mathbb{R}to mathbb{R}$ such that $f(1)=1$ and for all real numbers $x$ and $y$ with $y neq 0$, $$fBigg (yf(x)+frac{x}{y}Bigg)=xyf(x^2+y^2)$$




This seems quite hard. $f(x)=begin{cases}frac{1}{x}, xneq 0 \ 0, x=0 end{cases}$ works by inspection, as does $f(x)=0$ (though I'm not sure if this is legitimate. If we set $y=1$, we have $f(f(x)+x)=xf(x^2+1)$. If we set $x=1$, we have $f(frac{y^2+1}{y})=yf(y^2+1)$. which seems to imply $f(f(x)+x)=f(frac{x^2+1}{x})$. If I could show that $f$ is injective I could go further, but I'm stuck - subbing in other values doesn't really seem to lead anywhere either.



I believe this problem came from an Olympiad camp.










share|cite|improve this question











$endgroup$





Find all functions $f:mathbb{R}to mathbb{R}$ such that $f(1)=1$ and for all real numbers $x$ and $y$ with $y neq 0$, $$fBigg (yf(x)+frac{x}{y}Bigg)=xyf(x^2+y^2)$$




This seems quite hard. $f(x)=begin{cases}frac{1}{x}, xneq 0 \ 0, x=0 end{cases}$ works by inspection, as does $f(x)=0$ (though I'm not sure if this is legitimate. If we set $y=1$, we have $f(f(x)+x)=xf(x^2+1)$. If we set $x=1$, we have $f(frac{y^2+1}{y})=yf(y^2+1)$. which seems to imply $f(f(x)+x)=f(frac{x^2+1}{x})$. If I could show that $f$ is injective I could go further, but I'm stuck - subbing in other values doesn't really seem to lead anywhere either.



I believe this problem came from an Olympiad camp.







contest-math functional-equations






share|cite|improve this question















share|cite|improve this question













share|cite|improve this question




share|cite|improve this question








edited Mar 31 at 7:14









Eric Wofsey

192k14220352




192k14220352










asked Mar 21 at 5:25









user574848user574848

689118




689118












  • $begingroup$
    Are you sure it is $yneq 1$ and not $yneq 0$?
    $endgroup$
    – Redundant Aunt
    Mar 21 at 11:55










  • $begingroup$
    @RedundantAunt should be $yneq 0$ - fixed, thanks!
    $endgroup$
    – user574848
    Mar 21 at 11:59










  • $begingroup$
    The problem seems really hard to me! Are you sure that it has a nice solution or might there exist pathological solutions to this functional equation?
    $endgroup$
    – Redundant Aunt
    Mar 26 at 8:12










  • $begingroup$
    @RedundantAunt this question was shared to me - unfortunately, I can’t solve it either! (Hence why I’m asking)
    $endgroup$
    – user574848
    Mar 26 at 11:35








  • 3




    $begingroup$
    This is a duplicate of Solving for the implicit function $fleft(f(x)y+frac{x}{y}right)=xyfleft(x^2+y^2right)$ and $f(1)=1$, and AOPS links artofproblemsolving.com/community/c6h411400p2923354 , artofproblemsolving.com/community/c6h148711 and artofproblemsolving.com/community/c6h358223. And the original source maa.org/sites/default/files/pdf/AMC/a-activities/a6-mosp/… (Problem 7.4)
    $endgroup$
    – Sil
    Mar 31 at 11:10


















  • $begingroup$
    Are you sure it is $yneq 1$ and not $yneq 0$?
    $endgroup$
    – Redundant Aunt
    Mar 21 at 11:55










  • $begingroup$
    @RedundantAunt should be $yneq 0$ - fixed, thanks!
    $endgroup$
    – user574848
    Mar 21 at 11:59










  • $begingroup$
    The problem seems really hard to me! Are you sure that it has a nice solution or might there exist pathological solutions to this functional equation?
    $endgroup$
    – Redundant Aunt
    Mar 26 at 8:12










  • $begingroup$
    @RedundantAunt this question was shared to me - unfortunately, I can’t solve it either! (Hence why I’m asking)
    $endgroup$
    – user574848
    Mar 26 at 11:35








  • 3




    $begingroup$
    This is a duplicate of Solving for the implicit function $fleft(f(x)y+frac{x}{y}right)=xyfleft(x^2+y^2right)$ and $f(1)=1$, and AOPS links artofproblemsolving.com/community/c6h411400p2923354 , artofproblemsolving.com/community/c6h148711 and artofproblemsolving.com/community/c6h358223. And the original source maa.org/sites/default/files/pdf/AMC/a-activities/a6-mosp/… (Problem 7.4)
    $endgroup$
    – Sil
    Mar 31 at 11:10
















$begingroup$
Are you sure it is $yneq 1$ and not $yneq 0$?
$endgroup$
– Redundant Aunt
Mar 21 at 11:55




$begingroup$
Are you sure it is $yneq 1$ and not $yneq 0$?
$endgroup$
– Redundant Aunt
Mar 21 at 11:55












$begingroup$
@RedundantAunt should be $yneq 0$ - fixed, thanks!
$endgroup$
– user574848
Mar 21 at 11:59




$begingroup$
@RedundantAunt should be $yneq 0$ - fixed, thanks!
$endgroup$
– user574848
Mar 21 at 11:59












$begingroup$
The problem seems really hard to me! Are you sure that it has a nice solution or might there exist pathological solutions to this functional equation?
$endgroup$
– Redundant Aunt
Mar 26 at 8:12




$begingroup$
The problem seems really hard to me! Are you sure that it has a nice solution or might there exist pathological solutions to this functional equation?
$endgroup$
– Redundant Aunt
Mar 26 at 8:12












$begingroup$
@RedundantAunt this question was shared to me - unfortunately, I can’t solve it either! (Hence why I’m asking)
$endgroup$
– user574848
Mar 26 at 11:35






$begingroup$
@RedundantAunt this question was shared to me - unfortunately, I can’t solve it either! (Hence why I’m asking)
$endgroup$
– user574848
Mar 26 at 11:35






3




3




$begingroup$
This is a duplicate of Solving for the implicit function $fleft(f(x)y+frac{x}{y}right)=xyfleft(x^2+y^2right)$ and $f(1)=1$, and AOPS links artofproblemsolving.com/community/c6h411400p2923354 , artofproblemsolving.com/community/c6h148711 and artofproblemsolving.com/community/c6h358223. And the original source maa.org/sites/default/files/pdf/AMC/a-activities/a6-mosp/… (Problem 7.4)
$endgroup$
– Sil
Mar 31 at 11:10




$begingroup$
This is a duplicate of Solving for the implicit function $fleft(f(x)y+frac{x}{y}right)=xyfleft(x^2+y^2right)$ and $f(1)=1$, and AOPS links artofproblemsolving.com/community/c6h411400p2923354 , artofproblemsolving.com/community/c6h148711 and artofproblemsolving.com/community/c6h358223. And the original source maa.org/sites/default/files/pdf/AMC/a-activities/a6-mosp/… (Problem 7.4)
$endgroup$
– Sil
Mar 31 at 11:10










4 Answers
4






active

oldest

votes


















5












$begingroup$

Here is one approach,



the equation $y f(x)+ frac{x}{y}=x^2 + y^2$ for $x neq 0$ does have at least one real root, since the equation is equivalent to a cubic equation. Denote this root by $lambda$.



inputting $lambda$ into the equation we find $f(x^2+y^2)=x lambda f(x^2+y^2)$. Now, notice that $x^2+y^2 neq 0$, if you can show that for some $sigma$ that $f(sigma)=0, iff sigma =0$ then the result folllows since, then $lambda = frac{1}{x}$ and form the initial functional equation you would obtain $f(x)lambda + frac{x}{lambda}=x^2+lambda^2, implies f(x)=frac{1}{x}$



Thus, the solution to the problem is found by asserting that you can prove that




$f(sigma)=0 , iff sigma = 0$




The road to glory I belive requires analysis of the following relation




$f(f(x)+x) = xf(x^2+1)$




Allows us to show that for $x neq 0$ then $f(x)=0 implies f(x^2+1)=0$ which implies $exists,$ a sequence $S_n to infty$ such that $f(S_n)=0$. For the following, assume $f$ is continuous.



Now, take $x^2+y^2=S_n$, the initial relation implies that
$$fleft(f(x)y+ frac{x}{y}right)=0, forall ,x^2+y^2=S_n$$



Now, for $y to 0^+$ and $x>0$ one finds that $f(x)y+frac{x}{y} to infty$ as $x to sqrt{S_n}$, thus $f$ takes zero values in the neighbourhood of $infty$.



Since $f(y+frac{1}{y}) = y f(1+y^2), implies f(x)=-f(-x), forall, |x| geq 2$ we find the same result in the neighbourhood of $- infty$. Notice now that for $y>1$ that $y+frac{1}{y} > 1+y^2$ one can translate the zeroes in the neighbourhood of $infty$ until you reach $2$. So that




$f(x)=0$ on $(-infty, -2], cup , [2, infty)$




Not sure where else to go from here, but this may provide a useful aid to a full solution.






share|cite|improve this answer









$endgroup$





















    3












    $begingroup$

    Summary: This solution shows that if a function $f:mathbb{R}tomathbb{R}$ satisfies $f(1)=1$ and $fleft(yf(x)+frac{x}{y}right)=xyfleft(x^2+y^2right)$ for all $x,yinmathbb{R},yneq0$, then $f(0)=0$ and $f(x)=frac{1}{x}$ for all $xneq0$. No additional assumptions on $f$ are necessary!



    Thanks @Sil for giving all these references! I meanwhile came up with a solution myself, and last but not least because not many solutions of this problem seem to be around, I would like to share mine.



    Suppose $f$ is a solution to this functional equation and for $x,yinmathbb{R}, yneq0$ write $P(x,y)$ for the assertion $fleft(yf(x)+frac{x}{y}right)=xyfleft(x^2+y^2right)$. Furthermore, we define $mathcal{N}:={x>0 | f(x)=0}$, and assume that $mathcal{N}neqemptyset$.



    As @Kevin already pointed out, we have $alphainmathcal{N}impliesalpha^2+1inmathcal{N}$, and in particular $mathcal{N}$ is unbounded. Furthermore, $P(1,alpha)$ gives also $alphainmathcal{N}impliesalpha+frac{1}{alpha}inmathcal{N}$. Now, if $alpha,betainmathcal{N}$ then
    $$
    P(alpha,beta):quad fleft(frac{alpha}{beta}right)=alphabeta fleft(alpha^2+beta^2right)\
    P(beta,alpha):quad fleft(frac{beta}{alpha}right)=betaalpha fleft(beta^2+alpha^2right)
    $$

    and thus $fleft(frac{alpha}{beta}right)=fleft(frac{beta}{alpha}right)$. Therefore, if $alphainmathcal{N}$ then
    $$
    fleft(frac{1}{alpha}right)=fleft(frac{alpha+frac{1}{alpha}}{alpha^2+1}right)=fleft(frac{alpha^2+1}{alpha+frac{1}{alpha}}right)=f(alpha)=0
    $$

    and thus also $frac{1}{alpha}inmathcal{N}$. This gives, together with the unboundedness of $mathcal{N}$, the existence of $(alpha_n)_{ninmathbb{N}}inmathcal{N}^mathbb{N}$ with $lim_{ntoinfty}alpha_n=0$.



    Now notice that for $xneq 0$ and $alphainmathcal{N}$
    $$
    P(alpha,alpha^2 x):quad fleft(frac{1}{alpha x}right)=alpha^3 x fleft(alpha^4left(x^2+frac{1}{alpha^2}right)right)\
    P(frac{1}{alpha}, x):quad fleft(frac{1}{alpha x}right)=frac{x}{alpha} fleft(x^2+frac{1}{alpha^2}right)
    $$

    and thus $alpha^4 fleft(alpha^4left(x^2+frac{1}{alpha^2}right)right)=fleft(x^2+frac{1}{alpha^2}right)$ for all $xneq 0$, or in more simple terms
    $$
    (*)qquadalpha^4 f(alpha^4 z) = f(z) quadforall alphainmathcal{N}, z>frac{1}{alpha^2}
    $$

    Now for a fixed $alphainmathcal{N}$ and $y>0$ let $ninmathbb{N}$ be such that $y>max{frac{alpha_n^2}{alpha^4},frac{alpha_n^4}{alpha^2},alpha_n^2}$, which exists as $(alpha_n)to 0$. Then
    $$
    alpha^4 f(alpha^4 y)=frac{alpha^4}{alpha_n^4} alpha_n^4 fleft(alpha_n^4frac{alpha^4 y}{alpha_n^4}right)overset{frac{alpha^4 y}{alpha_n^4}>frac{1}{alpha_n^2}}{=}frac{1}{alpha_n^4}alpha^4 fleft(alpha^4frac{y}{alpha_n^4}right)overset{frac{y}{alpha_n^4}>frac{1}{alpha^2}}{=}frac{1}{alpha_n^4}fleft(frac{y}{alpha_n^4}right)overset{y>alpha_n^2}{=}f(y).
    $$

    Thus we can strengthen $(*)$ and actually have
    $$
    (**)qquadalpha^4 f(alpha^4 z) = f(z) quadforall alphainmathcal{N}, z>0.
    $$

    In particular, for $z=frac{1}{alpha^2}$ we get $alpha^4f(alpha^2)=fleft(frac{1}{alpha^2}right)$. On the other hand, we see that as $1+alpha^2,1+frac{1}{alpha^2}inmathcal{N}$, we have by again combining $P(1+alpha^2,1+frac{1}{alpha^2})$ and $P(1+frac{1}{alpha^2},1+alpha^2)$ that
    $$
    f(alpha^2)=fleft(frac{1+alpha^2}{1+frac{1}{alpha^2}}right)=fleft(frac{1+frac{1}{alpha^2}}{1+alpha^2}right)=fleft(frac{1}{alpha^2}right)
    $$

    and thus, as $alpha>0$ and $alphaneq 1$, we have $alpha^4f(alpha^2)=fleft(frac{1}{alpha^2}right)=f(alpha^2)implies f(alpha^2)=0$ so $alpha^2inmathcal{N}$. But then also $alpha^4inmathcal{N}$ which gives by $(**)$
    $$
    0=alpha^4 f(alpha^4)overset{(**)}{=} f(1)=1,
    $$

    contradiction!



    Therefore we conclude that $mathcal{N}$ is empty, and as @Kevin already saw this gives $f(x)=frac{1}{x}$ for all $xneq 0$. As $P(0,y)$ gives $f(0)=0$, we have uniquely determined $f$, and we can verify easily that $f$ is indeed a solution of the equation at hand.






    share|cite|improve this answer











    $endgroup$





















      0












      $begingroup$

      $color{brown}{textbf{Some forms of the equation.}}$



      If $underline{xnot=0},$ then unknowns can be swapped. So
      $$fleft(yf(x)+dfrac xyright) = xyf(x^2+y^2) = fleft(xf(y)+dfrac yxright),tag1$$
      with the partial cases
      $$begin{cases}
      y=1,quad f(f(x)+x) = xf(x^2+1) = fleft(x+dfrac1xright) hspace{226mu}(2.1)\[4pt]
      y=x,quad f(xf(x)+1) = x^2f(2x^2) hspace{350mu}(2.2)\[4pt]
      end{cases}$$

      Denote
      $$g(x) = xf(x)tag3,$$
      then from $(2)$ should
      $$begin{cases}
      g(1+x^2) = gleft(x+dfrac1xright) hspace{432mu}(4.1)\[4pt]
      g(g(x)+1) = frac12g(2x^2)(g(x)+1)hspace{370mu}(4.2)\[4pt]
      end{cases}$$

      Assume $g(x)$ continuous function.



      $color{brown}{textbf{Corollaries from the formula (4.1).}}$



      Using the relationships between the arguments in $(4.1)$ in the form of
      begin{cases}
      L_{1,2}=1+x^2=dfrac 12R(Rpmsqrt{R^2-4})\[4pt]
      R_{1,2}=x+dfrac1x = pmleft(sqrt {L-1}+dfrac1{sqrt{L-1}}right),
      end{cases}

      one can present equation $(4.1)$ in the forms of
      begin{cases}
      g(x) = gleft(dfrac 12x(xpmsqrt{x^2-4})right)hspace{382mu}(5.1)\[4pt]
      g(x) = gleft(pmleft(sqrt{x-1} + dfrac1{sqrt{x-1}}right)right).hspace{330mu}(5.2)
      end{cases}

      From $(5.2)$ should $g(1)=g(pminfty),$
      $$g(pminfty)=1.$$
      Also, formula $(5.2)$ allows to assign to each point of the interval $(1,2)$ the point of the interval $(2,infty)$ with the same value of the function $g.$



      At the same time, repeated recursive application of formula $(5.1)$ allows to prove that
      $$g(x)=1quad forall quad xin((-infty,-2]cup[2,infty)).tag6$$



      $color{brown}{textbf{Corollaries from the formula (4.2).}}$



      Let us consider such neighbour of the point $x=1,$ where $g(x)>0.$
      Then the right part of the system $(4.2)$ can be presented in the form of
      $$g(2x^2)(1+g(x)) = 2.tag7$$
      Applying $(7)$ for $x$ from $1$ to $+0$ and from $-2$ to $-0,$ easy to see that
      $g(x)=1.tag8$



      The value in the singular point $x=0$ can be defined immediately from the equaion $(1)$ and equals to zero.



      Theerefore, the OP solution
      $$f(x) =
      begin{cases}
      0,quadtext{if}quad x=0\[4pt]
      dfrac1x,quadtext{otherwize}
      end{cases}$$

      is the single non-trivial solution.






      share|cite|improve this answer











      $endgroup$













      • $begingroup$
        How do you conclude that $f(-y)=f(y)$ from $(1)$?
        $endgroup$
        – Servaes
        Mar 29 at 21:49










      • $begingroup$
        Also, how do you account for the solution $f=0$?
        $endgroup$
        – Servaes
        Mar 29 at 21:54










      • $begingroup$
        In fact, if $f$ is monotonic it is injective and then the fact that $$f(yf(x)+tfrac{x}{y})=xyf(x^2+y^2)=(-x)(-y)f((-x)^2+(-y)^2)=f(-yf(-x)+tfrac{-x}{-y}),$$ shows that $f(-x)=-f(x)$ for all $xinBbb{R}$, directly contradicting $(2)$ unless $f=0$.
        $endgroup$
        – Servaes
        Mar 29 at 22:00










      • $begingroup$
        Also, I don't see immediately how you prove that $f$ is monotonic on these intervals; your observation partitions $(1,infty)$ into infinite sequences on which $f$ is monotonic, but I don't see how this implies that $f$ is monotonic on the entire interval. The same for the interval $(0,1)$.
        $endgroup$
        – Servaes
        Mar 29 at 22:06












      • $begingroup$
        @Servaes Thanks! Fixed all.
        $endgroup$
        – Yuri Negometyanov
        Apr 1 at 19:17



















      0












      $begingroup$

      Let $f:textbf{R}rightarrowtextbf{R}$, such that $f(1)=1$ and
      $$
      fleft(yf(x)+frac{x}{y}right)=xyf(x^2+y^2)textrm{, }forall (x,y)intextbf{R}timestextbf{R}^{*}tag 1
      $$

      Set $x=0$, $y=1$ in (1), then easily $f(0)=0$ (the case $f(0)neq 0$ is trivial). For $y=1$ in (1) we get
      $$
      xf(x^2+1)=f(f(x)+x)textrm{, }xneq 0.tag 2
      $$

      Also using the symmetry we get
      $$
      fleft(yf(x)+frac{x}{y}right)=fleft(xf(y)+frac{y}{x}right)textrm{, }x,yneq 0
      $$

      For $y=1$, we get
      $$
      fleft(x+frac{1}{x}right)=f(f(x)+x)textrm{, }xneq 0tag 3
      $$

      Assume that $h(x,y)$ is a surface (function) such that
      $$
      f(y+h(x,y))=f(x).tag 4
      $$

      Note.



      One choise of $h(x,y)$ is $h(x,y)=-y+x$ but may be there others. For example if $f(x)=x^2-x+1$, then $h(x,y)=x-y$ or $h(x,y)=1-x-y$.



      We assume here that
      $$
      f(x)=hleft(x+frac{1}{x},xright)tag{4.1}
      $$


      From (4) with $xrightarrow x+frac{1}{x}$ we get
      $$
      fleft(x+frac{1}{x}right)=fleft(y+hleft(x+frac{1}{x},yright)right).
      $$

      Hence for $y=x$ in the above identity we get
      $$
      fleft(x+frac{1}{x}right)=fleft(x+hleft(x+frac{1}{x},xright)right)=f(x+f(x))
      $$

      Hence we can get (3). From (2) and (3) we get also
      $$
      xf(x^2+1)=fleft(x+frac{1}{x}right)tag 6
      $$

      Setting $xrightarrow x^{-1}$ in (6)
      $$
      frac{1}{x}fleft(1+frac{1}{x^2}right)=fleft(x+frac{1}{x}right)=xf(x^2+1).tag 7
      $$

      Hence if we set $x^2rightarrow x>0$ in (7), then
      $$
      fleft(1+frac{1}{x}right)=xf(x+1)textrm{, for all }x>0.tag 8
      $$

      Set also $x=y-1>0$ in (8), then
      $$
      fleft(frac{y}{y-1}right)=(y-1)f(y)textrm{, }y>1.
      $$

      With $y=1/w>1$ we get
      $$
      frac{1}{1-w}fleft(frac{1}{1-w}right)=frac{1}{w}fleft(frac{1}{w}right).
      $$

      Hence if $0<w<1$ and $g(w):=frac{1}{w}fleft(frac{1}{w}right)$, then
      $$
      f(x)=x^{-1}gleft(x^{-1}right)textrm{, where }x>1textrm{ and }g(1-w)=g(w)textrm{, }0<w<1tag 9
      $$

      The solution (9) satisfies (8),(7),(6), but for to holds (3) we get plus a new functional equation for $f(x)$ and this is (4.1).



      Hence the general solution is
      $$
      f(x)=hleft(x+frac{1}{x},xright)tag{11}
      $$

      with
      $$
      frac{1}{x-1}hleft(x-1+frac{1}{x-1},frac{1}{x-1}right)=frac{1}{x}hleft(x+frac{1}{x},frac{1}{x}right)tag{12}
      $$

      and $h(x,y)$ solution of
      $$
      f(y+h(x,y))=f(x).tag{13}
      $$

      An obvious solution of $(13)$ is $h(x,y)=x-y$, but it may exist and other solutions.






      share|cite|improve this answer









      $endgroup$














        Your Answer





        StackExchange.ifUsing("editor", function () {
        return StackExchange.using("mathjaxEditing", function () {
        StackExchange.MarkdownEditor.creationCallbacks.add(function (editor, postfix) {
        StackExchange.mathjaxEditing.prepareWmdForMathJax(editor, postfix, [["$", "$"], ["\\(","\\)"]]);
        });
        });
        }, "mathjax-editing");

        StackExchange.ready(function() {
        var channelOptions = {
        tags: "".split(" "),
        id: "69"
        };
        initTagRenderer("".split(" "), "".split(" "), channelOptions);

        StackExchange.using("externalEditor", function() {
        // Have to fire editor after snippets, if snippets enabled
        if (StackExchange.settings.snippets.snippetsEnabled) {
        StackExchange.using("snippets", function() {
        createEditor();
        });
        }
        else {
        createEditor();
        }
        });

        function createEditor() {
        StackExchange.prepareEditor({
        heartbeatType: 'answer',
        autoActivateHeartbeat: false,
        convertImagesToLinks: true,
        noModals: true,
        showLowRepImageUploadWarning: true,
        reputationToPostImages: 10,
        bindNavPrevention: true,
        postfix: "",
        imageUploader: {
        brandingHtml: "Powered by u003ca class="icon-imgur-white" href="https://imgur.com/"u003eu003c/au003e",
        contentPolicyHtml: "User contributions licensed under u003ca href="https://creativecommons.org/licenses/by-sa/3.0/"u003ecc by-sa 3.0 with attribution requiredu003c/au003e u003ca href="https://stackoverflow.com/legal/content-policy"u003e(content policy)u003c/au003e",
        allowUrls: true
        },
        noCode: true, onDemand: true,
        discardSelector: ".discard-answer"
        ,immediatelyShowMarkdownHelp:true
        });


        }
        });














        draft saved

        draft discarded


















        StackExchange.ready(
        function () {
        StackExchange.openid.initPostLogin('.new-post-login', 'https%3a%2f%2fmath.stackexchange.com%2fquestions%2f3156382%2fsolving-fyfxx-y-xyfx2y2-over-the-reals%23new-answer', 'question_page');
        }
        );

        Post as a guest















        Required, but never shown

























        4 Answers
        4






        active

        oldest

        votes








        4 Answers
        4






        active

        oldest

        votes









        active

        oldest

        votes






        active

        oldest

        votes









        5












        $begingroup$

        Here is one approach,



        the equation $y f(x)+ frac{x}{y}=x^2 + y^2$ for $x neq 0$ does have at least one real root, since the equation is equivalent to a cubic equation. Denote this root by $lambda$.



        inputting $lambda$ into the equation we find $f(x^2+y^2)=x lambda f(x^2+y^2)$. Now, notice that $x^2+y^2 neq 0$, if you can show that for some $sigma$ that $f(sigma)=0, iff sigma =0$ then the result folllows since, then $lambda = frac{1}{x}$ and form the initial functional equation you would obtain $f(x)lambda + frac{x}{lambda}=x^2+lambda^2, implies f(x)=frac{1}{x}$



        Thus, the solution to the problem is found by asserting that you can prove that




        $f(sigma)=0 , iff sigma = 0$




        The road to glory I belive requires analysis of the following relation




        $f(f(x)+x) = xf(x^2+1)$




        Allows us to show that for $x neq 0$ then $f(x)=0 implies f(x^2+1)=0$ which implies $exists,$ a sequence $S_n to infty$ such that $f(S_n)=0$. For the following, assume $f$ is continuous.



        Now, take $x^2+y^2=S_n$, the initial relation implies that
        $$fleft(f(x)y+ frac{x}{y}right)=0, forall ,x^2+y^2=S_n$$



        Now, for $y to 0^+$ and $x>0$ one finds that $f(x)y+frac{x}{y} to infty$ as $x to sqrt{S_n}$, thus $f$ takes zero values in the neighbourhood of $infty$.



        Since $f(y+frac{1}{y}) = y f(1+y^2), implies f(x)=-f(-x), forall, |x| geq 2$ we find the same result in the neighbourhood of $- infty$. Notice now that for $y>1$ that $y+frac{1}{y} > 1+y^2$ one can translate the zeroes in the neighbourhood of $infty$ until you reach $2$. So that




        $f(x)=0$ on $(-infty, -2], cup , [2, infty)$




        Not sure where else to go from here, but this may provide a useful aid to a full solution.






        share|cite|improve this answer









        $endgroup$


















          5












          $begingroup$

          Here is one approach,



          the equation $y f(x)+ frac{x}{y}=x^2 + y^2$ for $x neq 0$ does have at least one real root, since the equation is equivalent to a cubic equation. Denote this root by $lambda$.



          inputting $lambda$ into the equation we find $f(x^2+y^2)=x lambda f(x^2+y^2)$. Now, notice that $x^2+y^2 neq 0$, if you can show that for some $sigma$ that $f(sigma)=0, iff sigma =0$ then the result folllows since, then $lambda = frac{1}{x}$ and form the initial functional equation you would obtain $f(x)lambda + frac{x}{lambda}=x^2+lambda^2, implies f(x)=frac{1}{x}$



          Thus, the solution to the problem is found by asserting that you can prove that




          $f(sigma)=0 , iff sigma = 0$




          The road to glory I belive requires analysis of the following relation




          $f(f(x)+x) = xf(x^2+1)$




          Allows us to show that for $x neq 0$ then $f(x)=0 implies f(x^2+1)=0$ which implies $exists,$ a sequence $S_n to infty$ such that $f(S_n)=0$. For the following, assume $f$ is continuous.



          Now, take $x^2+y^2=S_n$, the initial relation implies that
          $$fleft(f(x)y+ frac{x}{y}right)=0, forall ,x^2+y^2=S_n$$



          Now, for $y to 0^+$ and $x>0$ one finds that $f(x)y+frac{x}{y} to infty$ as $x to sqrt{S_n}$, thus $f$ takes zero values in the neighbourhood of $infty$.



          Since $f(y+frac{1}{y}) = y f(1+y^2), implies f(x)=-f(-x), forall, |x| geq 2$ we find the same result in the neighbourhood of $- infty$. Notice now that for $y>1$ that $y+frac{1}{y} > 1+y^2$ one can translate the zeroes in the neighbourhood of $infty$ until you reach $2$. So that




          $f(x)=0$ on $(-infty, -2], cup , [2, infty)$




          Not sure where else to go from here, but this may provide a useful aid to a full solution.






          share|cite|improve this answer









          $endgroup$
















            5












            5








            5





            $begingroup$

            Here is one approach,



            the equation $y f(x)+ frac{x}{y}=x^2 + y^2$ for $x neq 0$ does have at least one real root, since the equation is equivalent to a cubic equation. Denote this root by $lambda$.



            inputting $lambda$ into the equation we find $f(x^2+y^2)=x lambda f(x^2+y^2)$. Now, notice that $x^2+y^2 neq 0$, if you can show that for some $sigma$ that $f(sigma)=0, iff sigma =0$ then the result folllows since, then $lambda = frac{1}{x}$ and form the initial functional equation you would obtain $f(x)lambda + frac{x}{lambda}=x^2+lambda^2, implies f(x)=frac{1}{x}$



            Thus, the solution to the problem is found by asserting that you can prove that




            $f(sigma)=0 , iff sigma = 0$




            The road to glory I belive requires analysis of the following relation




            $f(f(x)+x) = xf(x^2+1)$




            Allows us to show that for $x neq 0$ then $f(x)=0 implies f(x^2+1)=0$ which implies $exists,$ a sequence $S_n to infty$ such that $f(S_n)=0$. For the following, assume $f$ is continuous.



            Now, take $x^2+y^2=S_n$, the initial relation implies that
            $$fleft(f(x)y+ frac{x}{y}right)=0, forall ,x^2+y^2=S_n$$



            Now, for $y to 0^+$ and $x>0$ one finds that $f(x)y+frac{x}{y} to infty$ as $x to sqrt{S_n}$, thus $f$ takes zero values in the neighbourhood of $infty$.



            Since $f(y+frac{1}{y}) = y f(1+y^2), implies f(x)=-f(-x), forall, |x| geq 2$ we find the same result in the neighbourhood of $- infty$. Notice now that for $y>1$ that $y+frac{1}{y} > 1+y^2$ one can translate the zeroes in the neighbourhood of $infty$ until you reach $2$. So that




            $f(x)=0$ on $(-infty, -2], cup , [2, infty)$




            Not sure where else to go from here, but this may provide a useful aid to a full solution.






            share|cite|improve this answer









            $endgroup$



            Here is one approach,



            the equation $y f(x)+ frac{x}{y}=x^2 + y^2$ for $x neq 0$ does have at least one real root, since the equation is equivalent to a cubic equation. Denote this root by $lambda$.



            inputting $lambda$ into the equation we find $f(x^2+y^2)=x lambda f(x^2+y^2)$. Now, notice that $x^2+y^2 neq 0$, if you can show that for some $sigma$ that $f(sigma)=0, iff sigma =0$ then the result folllows since, then $lambda = frac{1}{x}$ and form the initial functional equation you would obtain $f(x)lambda + frac{x}{lambda}=x^2+lambda^2, implies f(x)=frac{1}{x}$



            Thus, the solution to the problem is found by asserting that you can prove that




            $f(sigma)=0 , iff sigma = 0$




            The road to glory I belive requires analysis of the following relation




            $f(f(x)+x) = xf(x^2+1)$




            Allows us to show that for $x neq 0$ then $f(x)=0 implies f(x^2+1)=0$ which implies $exists,$ a sequence $S_n to infty$ such that $f(S_n)=0$. For the following, assume $f$ is continuous.



            Now, take $x^2+y^2=S_n$, the initial relation implies that
            $$fleft(f(x)y+ frac{x}{y}right)=0, forall ,x^2+y^2=S_n$$



            Now, for $y to 0^+$ and $x>0$ one finds that $f(x)y+frac{x}{y} to infty$ as $x to sqrt{S_n}$, thus $f$ takes zero values in the neighbourhood of $infty$.



            Since $f(y+frac{1}{y}) = y f(1+y^2), implies f(x)=-f(-x), forall, |x| geq 2$ we find the same result in the neighbourhood of $- infty$. Notice now that for $y>1$ that $y+frac{1}{y} > 1+y^2$ one can translate the zeroes in the neighbourhood of $infty$ until you reach $2$. So that




            $f(x)=0$ on $(-infty, -2], cup , [2, infty)$




            Not sure where else to go from here, but this may provide a useful aid to a full solution.







            share|cite|improve this answer












            share|cite|improve this answer



            share|cite|improve this answer










            answered Mar 21 at 12:40









            KevinKevin

            5,746823




            5,746823























                3












                $begingroup$

                Summary: This solution shows that if a function $f:mathbb{R}tomathbb{R}$ satisfies $f(1)=1$ and $fleft(yf(x)+frac{x}{y}right)=xyfleft(x^2+y^2right)$ for all $x,yinmathbb{R},yneq0$, then $f(0)=0$ and $f(x)=frac{1}{x}$ for all $xneq0$. No additional assumptions on $f$ are necessary!



                Thanks @Sil for giving all these references! I meanwhile came up with a solution myself, and last but not least because not many solutions of this problem seem to be around, I would like to share mine.



                Suppose $f$ is a solution to this functional equation and for $x,yinmathbb{R}, yneq0$ write $P(x,y)$ for the assertion $fleft(yf(x)+frac{x}{y}right)=xyfleft(x^2+y^2right)$. Furthermore, we define $mathcal{N}:={x>0 | f(x)=0}$, and assume that $mathcal{N}neqemptyset$.



                As @Kevin already pointed out, we have $alphainmathcal{N}impliesalpha^2+1inmathcal{N}$, and in particular $mathcal{N}$ is unbounded. Furthermore, $P(1,alpha)$ gives also $alphainmathcal{N}impliesalpha+frac{1}{alpha}inmathcal{N}$. Now, if $alpha,betainmathcal{N}$ then
                $$
                P(alpha,beta):quad fleft(frac{alpha}{beta}right)=alphabeta fleft(alpha^2+beta^2right)\
                P(beta,alpha):quad fleft(frac{beta}{alpha}right)=betaalpha fleft(beta^2+alpha^2right)
                $$

                and thus $fleft(frac{alpha}{beta}right)=fleft(frac{beta}{alpha}right)$. Therefore, if $alphainmathcal{N}$ then
                $$
                fleft(frac{1}{alpha}right)=fleft(frac{alpha+frac{1}{alpha}}{alpha^2+1}right)=fleft(frac{alpha^2+1}{alpha+frac{1}{alpha}}right)=f(alpha)=0
                $$

                and thus also $frac{1}{alpha}inmathcal{N}$. This gives, together with the unboundedness of $mathcal{N}$, the existence of $(alpha_n)_{ninmathbb{N}}inmathcal{N}^mathbb{N}$ with $lim_{ntoinfty}alpha_n=0$.



                Now notice that for $xneq 0$ and $alphainmathcal{N}$
                $$
                P(alpha,alpha^2 x):quad fleft(frac{1}{alpha x}right)=alpha^3 x fleft(alpha^4left(x^2+frac{1}{alpha^2}right)right)\
                P(frac{1}{alpha}, x):quad fleft(frac{1}{alpha x}right)=frac{x}{alpha} fleft(x^2+frac{1}{alpha^2}right)
                $$

                and thus $alpha^4 fleft(alpha^4left(x^2+frac{1}{alpha^2}right)right)=fleft(x^2+frac{1}{alpha^2}right)$ for all $xneq 0$, or in more simple terms
                $$
                (*)qquadalpha^4 f(alpha^4 z) = f(z) quadforall alphainmathcal{N}, z>frac{1}{alpha^2}
                $$

                Now for a fixed $alphainmathcal{N}$ and $y>0$ let $ninmathbb{N}$ be such that $y>max{frac{alpha_n^2}{alpha^4},frac{alpha_n^4}{alpha^2},alpha_n^2}$, which exists as $(alpha_n)to 0$. Then
                $$
                alpha^4 f(alpha^4 y)=frac{alpha^4}{alpha_n^4} alpha_n^4 fleft(alpha_n^4frac{alpha^4 y}{alpha_n^4}right)overset{frac{alpha^4 y}{alpha_n^4}>frac{1}{alpha_n^2}}{=}frac{1}{alpha_n^4}alpha^4 fleft(alpha^4frac{y}{alpha_n^4}right)overset{frac{y}{alpha_n^4}>frac{1}{alpha^2}}{=}frac{1}{alpha_n^4}fleft(frac{y}{alpha_n^4}right)overset{y>alpha_n^2}{=}f(y).
                $$

                Thus we can strengthen $(*)$ and actually have
                $$
                (**)qquadalpha^4 f(alpha^4 z) = f(z) quadforall alphainmathcal{N}, z>0.
                $$

                In particular, for $z=frac{1}{alpha^2}$ we get $alpha^4f(alpha^2)=fleft(frac{1}{alpha^2}right)$. On the other hand, we see that as $1+alpha^2,1+frac{1}{alpha^2}inmathcal{N}$, we have by again combining $P(1+alpha^2,1+frac{1}{alpha^2})$ and $P(1+frac{1}{alpha^2},1+alpha^2)$ that
                $$
                f(alpha^2)=fleft(frac{1+alpha^2}{1+frac{1}{alpha^2}}right)=fleft(frac{1+frac{1}{alpha^2}}{1+alpha^2}right)=fleft(frac{1}{alpha^2}right)
                $$

                and thus, as $alpha>0$ and $alphaneq 1$, we have $alpha^4f(alpha^2)=fleft(frac{1}{alpha^2}right)=f(alpha^2)implies f(alpha^2)=0$ so $alpha^2inmathcal{N}$. But then also $alpha^4inmathcal{N}$ which gives by $(**)$
                $$
                0=alpha^4 f(alpha^4)overset{(**)}{=} f(1)=1,
                $$

                contradiction!



                Therefore we conclude that $mathcal{N}$ is empty, and as @Kevin already saw this gives $f(x)=frac{1}{x}$ for all $xneq 0$. As $P(0,y)$ gives $f(0)=0$, we have uniquely determined $f$, and we can verify easily that $f$ is indeed a solution of the equation at hand.






                share|cite|improve this answer











                $endgroup$


















                  3












                  $begingroup$

                  Summary: This solution shows that if a function $f:mathbb{R}tomathbb{R}$ satisfies $f(1)=1$ and $fleft(yf(x)+frac{x}{y}right)=xyfleft(x^2+y^2right)$ for all $x,yinmathbb{R},yneq0$, then $f(0)=0$ and $f(x)=frac{1}{x}$ for all $xneq0$. No additional assumptions on $f$ are necessary!



                  Thanks @Sil for giving all these references! I meanwhile came up with a solution myself, and last but not least because not many solutions of this problem seem to be around, I would like to share mine.



                  Suppose $f$ is a solution to this functional equation and for $x,yinmathbb{R}, yneq0$ write $P(x,y)$ for the assertion $fleft(yf(x)+frac{x}{y}right)=xyfleft(x^2+y^2right)$. Furthermore, we define $mathcal{N}:={x>0 | f(x)=0}$, and assume that $mathcal{N}neqemptyset$.



                  As @Kevin already pointed out, we have $alphainmathcal{N}impliesalpha^2+1inmathcal{N}$, and in particular $mathcal{N}$ is unbounded. Furthermore, $P(1,alpha)$ gives also $alphainmathcal{N}impliesalpha+frac{1}{alpha}inmathcal{N}$. Now, if $alpha,betainmathcal{N}$ then
                  $$
                  P(alpha,beta):quad fleft(frac{alpha}{beta}right)=alphabeta fleft(alpha^2+beta^2right)\
                  P(beta,alpha):quad fleft(frac{beta}{alpha}right)=betaalpha fleft(beta^2+alpha^2right)
                  $$

                  and thus $fleft(frac{alpha}{beta}right)=fleft(frac{beta}{alpha}right)$. Therefore, if $alphainmathcal{N}$ then
                  $$
                  fleft(frac{1}{alpha}right)=fleft(frac{alpha+frac{1}{alpha}}{alpha^2+1}right)=fleft(frac{alpha^2+1}{alpha+frac{1}{alpha}}right)=f(alpha)=0
                  $$

                  and thus also $frac{1}{alpha}inmathcal{N}$. This gives, together with the unboundedness of $mathcal{N}$, the existence of $(alpha_n)_{ninmathbb{N}}inmathcal{N}^mathbb{N}$ with $lim_{ntoinfty}alpha_n=0$.



                  Now notice that for $xneq 0$ and $alphainmathcal{N}$
                  $$
                  P(alpha,alpha^2 x):quad fleft(frac{1}{alpha x}right)=alpha^3 x fleft(alpha^4left(x^2+frac{1}{alpha^2}right)right)\
                  P(frac{1}{alpha}, x):quad fleft(frac{1}{alpha x}right)=frac{x}{alpha} fleft(x^2+frac{1}{alpha^2}right)
                  $$

                  and thus $alpha^4 fleft(alpha^4left(x^2+frac{1}{alpha^2}right)right)=fleft(x^2+frac{1}{alpha^2}right)$ for all $xneq 0$, or in more simple terms
                  $$
                  (*)qquadalpha^4 f(alpha^4 z) = f(z) quadforall alphainmathcal{N}, z>frac{1}{alpha^2}
                  $$

                  Now for a fixed $alphainmathcal{N}$ and $y>0$ let $ninmathbb{N}$ be such that $y>max{frac{alpha_n^2}{alpha^4},frac{alpha_n^4}{alpha^2},alpha_n^2}$, which exists as $(alpha_n)to 0$. Then
                  $$
                  alpha^4 f(alpha^4 y)=frac{alpha^4}{alpha_n^4} alpha_n^4 fleft(alpha_n^4frac{alpha^4 y}{alpha_n^4}right)overset{frac{alpha^4 y}{alpha_n^4}>frac{1}{alpha_n^2}}{=}frac{1}{alpha_n^4}alpha^4 fleft(alpha^4frac{y}{alpha_n^4}right)overset{frac{y}{alpha_n^4}>frac{1}{alpha^2}}{=}frac{1}{alpha_n^4}fleft(frac{y}{alpha_n^4}right)overset{y>alpha_n^2}{=}f(y).
                  $$

                  Thus we can strengthen $(*)$ and actually have
                  $$
                  (**)qquadalpha^4 f(alpha^4 z) = f(z) quadforall alphainmathcal{N}, z>0.
                  $$

                  In particular, for $z=frac{1}{alpha^2}$ we get $alpha^4f(alpha^2)=fleft(frac{1}{alpha^2}right)$. On the other hand, we see that as $1+alpha^2,1+frac{1}{alpha^2}inmathcal{N}$, we have by again combining $P(1+alpha^2,1+frac{1}{alpha^2})$ and $P(1+frac{1}{alpha^2},1+alpha^2)$ that
                  $$
                  f(alpha^2)=fleft(frac{1+alpha^2}{1+frac{1}{alpha^2}}right)=fleft(frac{1+frac{1}{alpha^2}}{1+alpha^2}right)=fleft(frac{1}{alpha^2}right)
                  $$

                  and thus, as $alpha>0$ and $alphaneq 1$, we have $alpha^4f(alpha^2)=fleft(frac{1}{alpha^2}right)=f(alpha^2)implies f(alpha^2)=0$ so $alpha^2inmathcal{N}$. But then also $alpha^4inmathcal{N}$ which gives by $(**)$
                  $$
                  0=alpha^4 f(alpha^4)overset{(**)}{=} f(1)=1,
                  $$

                  contradiction!



                  Therefore we conclude that $mathcal{N}$ is empty, and as @Kevin already saw this gives $f(x)=frac{1}{x}$ for all $xneq 0$. As $P(0,y)$ gives $f(0)=0$, we have uniquely determined $f$, and we can verify easily that $f$ is indeed a solution of the equation at hand.






                  share|cite|improve this answer











                  $endgroup$
















                    3












                    3








                    3





                    $begingroup$

                    Summary: This solution shows that if a function $f:mathbb{R}tomathbb{R}$ satisfies $f(1)=1$ and $fleft(yf(x)+frac{x}{y}right)=xyfleft(x^2+y^2right)$ for all $x,yinmathbb{R},yneq0$, then $f(0)=0$ and $f(x)=frac{1}{x}$ for all $xneq0$. No additional assumptions on $f$ are necessary!



                    Thanks @Sil for giving all these references! I meanwhile came up with a solution myself, and last but not least because not many solutions of this problem seem to be around, I would like to share mine.



                    Suppose $f$ is a solution to this functional equation and for $x,yinmathbb{R}, yneq0$ write $P(x,y)$ for the assertion $fleft(yf(x)+frac{x}{y}right)=xyfleft(x^2+y^2right)$. Furthermore, we define $mathcal{N}:={x>0 | f(x)=0}$, and assume that $mathcal{N}neqemptyset$.



                    As @Kevin already pointed out, we have $alphainmathcal{N}impliesalpha^2+1inmathcal{N}$, and in particular $mathcal{N}$ is unbounded. Furthermore, $P(1,alpha)$ gives also $alphainmathcal{N}impliesalpha+frac{1}{alpha}inmathcal{N}$. Now, if $alpha,betainmathcal{N}$ then
                    $$
                    P(alpha,beta):quad fleft(frac{alpha}{beta}right)=alphabeta fleft(alpha^2+beta^2right)\
                    P(beta,alpha):quad fleft(frac{beta}{alpha}right)=betaalpha fleft(beta^2+alpha^2right)
                    $$

                    and thus $fleft(frac{alpha}{beta}right)=fleft(frac{beta}{alpha}right)$. Therefore, if $alphainmathcal{N}$ then
                    $$
                    fleft(frac{1}{alpha}right)=fleft(frac{alpha+frac{1}{alpha}}{alpha^2+1}right)=fleft(frac{alpha^2+1}{alpha+frac{1}{alpha}}right)=f(alpha)=0
                    $$

                    and thus also $frac{1}{alpha}inmathcal{N}$. This gives, together with the unboundedness of $mathcal{N}$, the existence of $(alpha_n)_{ninmathbb{N}}inmathcal{N}^mathbb{N}$ with $lim_{ntoinfty}alpha_n=0$.



                    Now notice that for $xneq 0$ and $alphainmathcal{N}$
                    $$
                    P(alpha,alpha^2 x):quad fleft(frac{1}{alpha x}right)=alpha^3 x fleft(alpha^4left(x^2+frac{1}{alpha^2}right)right)\
                    P(frac{1}{alpha}, x):quad fleft(frac{1}{alpha x}right)=frac{x}{alpha} fleft(x^2+frac{1}{alpha^2}right)
                    $$

                    and thus $alpha^4 fleft(alpha^4left(x^2+frac{1}{alpha^2}right)right)=fleft(x^2+frac{1}{alpha^2}right)$ for all $xneq 0$, or in more simple terms
                    $$
                    (*)qquadalpha^4 f(alpha^4 z) = f(z) quadforall alphainmathcal{N}, z>frac{1}{alpha^2}
                    $$

                    Now for a fixed $alphainmathcal{N}$ and $y>0$ let $ninmathbb{N}$ be such that $y>max{frac{alpha_n^2}{alpha^4},frac{alpha_n^4}{alpha^2},alpha_n^2}$, which exists as $(alpha_n)to 0$. Then
                    $$
                    alpha^4 f(alpha^4 y)=frac{alpha^4}{alpha_n^4} alpha_n^4 fleft(alpha_n^4frac{alpha^4 y}{alpha_n^4}right)overset{frac{alpha^4 y}{alpha_n^4}>frac{1}{alpha_n^2}}{=}frac{1}{alpha_n^4}alpha^4 fleft(alpha^4frac{y}{alpha_n^4}right)overset{frac{y}{alpha_n^4}>frac{1}{alpha^2}}{=}frac{1}{alpha_n^4}fleft(frac{y}{alpha_n^4}right)overset{y>alpha_n^2}{=}f(y).
                    $$

                    Thus we can strengthen $(*)$ and actually have
                    $$
                    (**)qquadalpha^4 f(alpha^4 z) = f(z) quadforall alphainmathcal{N}, z>0.
                    $$

                    In particular, for $z=frac{1}{alpha^2}$ we get $alpha^4f(alpha^2)=fleft(frac{1}{alpha^2}right)$. On the other hand, we see that as $1+alpha^2,1+frac{1}{alpha^2}inmathcal{N}$, we have by again combining $P(1+alpha^2,1+frac{1}{alpha^2})$ and $P(1+frac{1}{alpha^2},1+alpha^2)$ that
                    $$
                    f(alpha^2)=fleft(frac{1+alpha^2}{1+frac{1}{alpha^2}}right)=fleft(frac{1+frac{1}{alpha^2}}{1+alpha^2}right)=fleft(frac{1}{alpha^2}right)
                    $$

                    and thus, as $alpha>0$ and $alphaneq 1$, we have $alpha^4f(alpha^2)=fleft(frac{1}{alpha^2}right)=f(alpha^2)implies f(alpha^2)=0$ so $alpha^2inmathcal{N}$. But then also $alpha^4inmathcal{N}$ which gives by $(**)$
                    $$
                    0=alpha^4 f(alpha^4)overset{(**)}{=} f(1)=1,
                    $$

                    contradiction!



                    Therefore we conclude that $mathcal{N}$ is empty, and as @Kevin already saw this gives $f(x)=frac{1}{x}$ for all $xneq 0$. As $P(0,y)$ gives $f(0)=0$, we have uniquely determined $f$, and we can verify easily that $f$ is indeed a solution of the equation at hand.






                    share|cite|improve this answer











                    $endgroup$



                    Summary: This solution shows that if a function $f:mathbb{R}tomathbb{R}$ satisfies $f(1)=1$ and $fleft(yf(x)+frac{x}{y}right)=xyfleft(x^2+y^2right)$ for all $x,yinmathbb{R},yneq0$, then $f(0)=0$ and $f(x)=frac{1}{x}$ for all $xneq0$. No additional assumptions on $f$ are necessary!



                    Thanks @Sil for giving all these references! I meanwhile came up with a solution myself, and last but not least because not many solutions of this problem seem to be around, I would like to share mine.



                    Suppose $f$ is a solution to this functional equation and for $x,yinmathbb{R}, yneq0$ write $P(x,y)$ for the assertion $fleft(yf(x)+frac{x}{y}right)=xyfleft(x^2+y^2right)$. Furthermore, we define $mathcal{N}:={x>0 | f(x)=0}$, and assume that $mathcal{N}neqemptyset$.



                    As @Kevin already pointed out, we have $alphainmathcal{N}impliesalpha^2+1inmathcal{N}$, and in particular $mathcal{N}$ is unbounded. Furthermore, $P(1,alpha)$ gives also $alphainmathcal{N}impliesalpha+frac{1}{alpha}inmathcal{N}$. Now, if $alpha,betainmathcal{N}$ then
                    $$
                    P(alpha,beta):quad fleft(frac{alpha}{beta}right)=alphabeta fleft(alpha^2+beta^2right)\
                    P(beta,alpha):quad fleft(frac{beta}{alpha}right)=betaalpha fleft(beta^2+alpha^2right)
                    $$

                    and thus $fleft(frac{alpha}{beta}right)=fleft(frac{beta}{alpha}right)$. Therefore, if $alphainmathcal{N}$ then
                    $$
                    fleft(frac{1}{alpha}right)=fleft(frac{alpha+frac{1}{alpha}}{alpha^2+1}right)=fleft(frac{alpha^2+1}{alpha+frac{1}{alpha}}right)=f(alpha)=0
                    $$

                    and thus also $frac{1}{alpha}inmathcal{N}$. This gives, together with the unboundedness of $mathcal{N}$, the existence of $(alpha_n)_{ninmathbb{N}}inmathcal{N}^mathbb{N}$ with $lim_{ntoinfty}alpha_n=0$.



                    Now notice that for $xneq 0$ and $alphainmathcal{N}$
                    $$
                    P(alpha,alpha^2 x):quad fleft(frac{1}{alpha x}right)=alpha^3 x fleft(alpha^4left(x^2+frac{1}{alpha^2}right)right)\
                    P(frac{1}{alpha}, x):quad fleft(frac{1}{alpha x}right)=frac{x}{alpha} fleft(x^2+frac{1}{alpha^2}right)
                    $$

                    and thus $alpha^4 fleft(alpha^4left(x^2+frac{1}{alpha^2}right)right)=fleft(x^2+frac{1}{alpha^2}right)$ for all $xneq 0$, or in more simple terms
                    $$
                    (*)qquadalpha^4 f(alpha^4 z) = f(z) quadforall alphainmathcal{N}, z>frac{1}{alpha^2}
                    $$

                    Now for a fixed $alphainmathcal{N}$ and $y>0$ let $ninmathbb{N}$ be such that $y>max{frac{alpha_n^2}{alpha^4},frac{alpha_n^4}{alpha^2},alpha_n^2}$, which exists as $(alpha_n)to 0$. Then
                    $$
                    alpha^4 f(alpha^4 y)=frac{alpha^4}{alpha_n^4} alpha_n^4 fleft(alpha_n^4frac{alpha^4 y}{alpha_n^4}right)overset{frac{alpha^4 y}{alpha_n^4}>frac{1}{alpha_n^2}}{=}frac{1}{alpha_n^4}alpha^4 fleft(alpha^4frac{y}{alpha_n^4}right)overset{frac{y}{alpha_n^4}>frac{1}{alpha^2}}{=}frac{1}{alpha_n^4}fleft(frac{y}{alpha_n^4}right)overset{y>alpha_n^2}{=}f(y).
                    $$

                    Thus we can strengthen $(*)$ and actually have
                    $$
                    (**)qquadalpha^4 f(alpha^4 z) = f(z) quadforall alphainmathcal{N}, z>0.
                    $$

                    In particular, for $z=frac{1}{alpha^2}$ we get $alpha^4f(alpha^2)=fleft(frac{1}{alpha^2}right)$. On the other hand, we see that as $1+alpha^2,1+frac{1}{alpha^2}inmathcal{N}$, we have by again combining $P(1+alpha^2,1+frac{1}{alpha^2})$ and $P(1+frac{1}{alpha^2},1+alpha^2)$ that
                    $$
                    f(alpha^2)=fleft(frac{1+alpha^2}{1+frac{1}{alpha^2}}right)=fleft(frac{1+frac{1}{alpha^2}}{1+alpha^2}right)=fleft(frac{1}{alpha^2}right)
                    $$

                    and thus, as $alpha>0$ and $alphaneq 1$, we have $alpha^4f(alpha^2)=fleft(frac{1}{alpha^2}right)=f(alpha^2)implies f(alpha^2)=0$ so $alpha^2inmathcal{N}$. But then also $alpha^4inmathcal{N}$ which gives by $(**)$
                    $$
                    0=alpha^4 f(alpha^4)overset{(**)}{=} f(1)=1,
                    $$

                    contradiction!



                    Therefore we conclude that $mathcal{N}$ is empty, and as @Kevin already saw this gives $f(x)=frac{1}{x}$ for all $xneq 0$. As $P(0,y)$ gives $f(0)=0$, we have uniquely determined $f$, and we can verify easily that $f$ is indeed a solution of the equation at hand.







                    share|cite|improve this answer














                    share|cite|improve this answer



                    share|cite|improve this answer








                    edited Apr 2 at 7:09

























                    answered Apr 1 at 13:20









                    Redundant AuntRedundant Aunt

                    7,22121244




                    7,22121244























                        0












                        $begingroup$

                        $color{brown}{textbf{Some forms of the equation.}}$



                        If $underline{xnot=0},$ then unknowns can be swapped. So
                        $$fleft(yf(x)+dfrac xyright) = xyf(x^2+y^2) = fleft(xf(y)+dfrac yxright),tag1$$
                        with the partial cases
                        $$begin{cases}
                        y=1,quad f(f(x)+x) = xf(x^2+1) = fleft(x+dfrac1xright) hspace{226mu}(2.1)\[4pt]
                        y=x,quad f(xf(x)+1) = x^2f(2x^2) hspace{350mu}(2.2)\[4pt]
                        end{cases}$$

                        Denote
                        $$g(x) = xf(x)tag3,$$
                        then from $(2)$ should
                        $$begin{cases}
                        g(1+x^2) = gleft(x+dfrac1xright) hspace{432mu}(4.1)\[4pt]
                        g(g(x)+1) = frac12g(2x^2)(g(x)+1)hspace{370mu}(4.2)\[4pt]
                        end{cases}$$

                        Assume $g(x)$ continuous function.



                        $color{brown}{textbf{Corollaries from the formula (4.1).}}$



                        Using the relationships between the arguments in $(4.1)$ in the form of
                        begin{cases}
                        L_{1,2}=1+x^2=dfrac 12R(Rpmsqrt{R^2-4})\[4pt]
                        R_{1,2}=x+dfrac1x = pmleft(sqrt {L-1}+dfrac1{sqrt{L-1}}right),
                        end{cases}

                        one can present equation $(4.1)$ in the forms of
                        begin{cases}
                        g(x) = gleft(dfrac 12x(xpmsqrt{x^2-4})right)hspace{382mu}(5.1)\[4pt]
                        g(x) = gleft(pmleft(sqrt{x-1} + dfrac1{sqrt{x-1}}right)right).hspace{330mu}(5.2)
                        end{cases}

                        From $(5.2)$ should $g(1)=g(pminfty),$
                        $$g(pminfty)=1.$$
                        Also, formula $(5.2)$ allows to assign to each point of the interval $(1,2)$ the point of the interval $(2,infty)$ with the same value of the function $g.$



                        At the same time, repeated recursive application of formula $(5.1)$ allows to prove that
                        $$g(x)=1quad forall quad xin((-infty,-2]cup[2,infty)).tag6$$



                        $color{brown}{textbf{Corollaries from the formula (4.2).}}$



                        Let us consider such neighbour of the point $x=1,$ where $g(x)>0.$
                        Then the right part of the system $(4.2)$ can be presented in the form of
                        $$g(2x^2)(1+g(x)) = 2.tag7$$
                        Applying $(7)$ for $x$ from $1$ to $+0$ and from $-2$ to $-0,$ easy to see that
                        $g(x)=1.tag8$



                        The value in the singular point $x=0$ can be defined immediately from the equaion $(1)$ and equals to zero.



                        Theerefore, the OP solution
                        $$f(x) =
                        begin{cases}
                        0,quadtext{if}quad x=0\[4pt]
                        dfrac1x,quadtext{otherwize}
                        end{cases}$$

                        is the single non-trivial solution.






                        share|cite|improve this answer











                        $endgroup$













                        • $begingroup$
                          How do you conclude that $f(-y)=f(y)$ from $(1)$?
                          $endgroup$
                          – Servaes
                          Mar 29 at 21:49










                        • $begingroup$
                          Also, how do you account for the solution $f=0$?
                          $endgroup$
                          – Servaes
                          Mar 29 at 21:54










                        • $begingroup$
                          In fact, if $f$ is monotonic it is injective and then the fact that $$f(yf(x)+tfrac{x}{y})=xyf(x^2+y^2)=(-x)(-y)f((-x)^2+(-y)^2)=f(-yf(-x)+tfrac{-x}{-y}),$$ shows that $f(-x)=-f(x)$ for all $xinBbb{R}$, directly contradicting $(2)$ unless $f=0$.
                          $endgroup$
                          – Servaes
                          Mar 29 at 22:00










                        • $begingroup$
                          Also, I don't see immediately how you prove that $f$ is monotonic on these intervals; your observation partitions $(1,infty)$ into infinite sequences on which $f$ is monotonic, but I don't see how this implies that $f$ is monotonic on the entire interval. The same for the interval $(0,1)$.
                          $endgroup$
                          – Servaes
                          Mar 29 at 22:06












                        • $begingroup$
                          @Servaes Thanks! Fixed all.
                          $endgroup$
                          – Yuri Negometyanov
                          Apr 1 at 19:17
















                        0












                        $begingroup$

                        $color{brown}{textbf{Some forms of the equation.}}$



                        If $underline{xnot=0},$ then unknowns can be swapped. So
                        $$fleft(yf(x)+dfrac xyright) = xyf(x^2+y^2) = fleft(xf(y)+dfrac yxright),tag1$$
                        with the partial cases
                        $$begin{cases}
                        y=1,quad f(f(x)+x) = xf(x^2+1) = fleft(x+dfrac1xright) hspace{226mu}(2.1)\[4pt]
                        y=x,quad f(xf(x)+1) = x^2f(2x^2) hspace{350mu}(2.2)\[4pt]
                        end{cases}$$

                        Denote
                        $$g(x) = xf(x)tag3,$$
                        then from $(2)$ should
                        $$begin{cases}
                        g(1+x^2) = gleft(x+dfrac1xright) hspace{432mu}(4.1)\[4pt]
                        g(g(x)+1) = frac12g(2x^2)(g(x)+1)hspace{370mu}(4.2)\[4pt]
                        end{cases}$$

                        Assume $g(x)$ continuous function.



                        $color{brown}{textbf{Corollaries from the formula (4.1).}}$



                        Using the relationships between the arguments in $(4.1)$ in the form of
                        begin{cases}
                        L_{1,2}=1+x^2=dfrac 12R(Rpmsqrt{R^2-4})\[4pt]
                        R_{1,2}=x+dfrac1x = pmleft(sqrt {L-1}+dfrac1{sqrt{L-1}}right),
                        end{cases}

                        one can present equation $(4.1)$ in the forms of
                        begin{cases}
                        g(x) = gleft(dfrac 12x(xpmsqrt{x^2-4})right)hspace{382mu}(5.1)\[4pt]
                        g(x) = gleft(pmleft(sqrt{x-1} + dfrac1{sqrt{x-1}}right)right).hspace{330mu}(5.2)
                        end{cases}

                        From $(5.2)$ should $g(1)=g(pminfty),$
                        $$g(pminfty)=1.$$
                        Also, formula $(5.2)$ allows to assign to each point of the interval $(1,2)$ the point of the interval $(2,infty)$ with the same value of the function $g.$



                        At the same time, repeated recursive application of formula $(5.1)$ allows to prove that
                        $$g(x)=1quad forall quad xin((-infty,-2]cup[2,infty)).tag6$$



                        $color{brown}{textbf{Corollaries from the formula (4.2).}}$



                        Let us consider such neighbour of the point $x=1,$ where $g(x)>0.$
                        Then the right part of the system $(4.2)$ can be presented in the form of
                        $$g(2x^2)(1+g(x)) = 2.tag7$$
                        Applying $(7)$ for $x$ from $1$ to $+0$ and from $-2$ to $-0,$ easy to see that
                        $g(x)=1.tag8$



                        The value in the singular point $x=0$ can be defined immediately from the equaion $(1)$ and equals to zero.



                        Theerefore, the OP solution
                        $$f(x) =
                        begin{cases}
                        0,quadtext{if}quad x=0\[4pt]
                        dfrac1x,quadtext{otherwize}
                        end{cases}$$

                        is the single non-trivial solution.






                        share|cite|improve this answer











                        $endgroup$













                        • $begingroup$
                          How do you conclude that $f(-y)=f(y)$ from $(1)$?
                          $endgroup$
                          – Servaes
                          Mar 29 at 21:49










                        • $begingroup$
                          Also, how do you account for the solution $f=0$?
                          $endgroup$
                          – Servaes
                          Mar 29 at 21:54










                        • $begingroup$
                          In fact, if $f$ is monotonic it is injective and then the fact that $$f(yf(x)+tfrac{x}{y})=xyf(x^2+y^2)=(-x)(-y)f((-x)^2+(-y)^2)=f(-yf(-x)+tfrac{-x}{-y}),$$ shows that $f(-x)=-f(x)$ for all $xinBbb{R}$, directly contradicting $(2)$ unless $f=0$.
                          $endgroup$
                          – Servaes
                          Mar 29 at 22:00










                        • $begingroup$
                          Also, I don't see immediately how you prove that $f$ is monotonic on these intervals; your observation partitions $(1,infty)$ into infinite sequences on which $f$ is monotonic, but I don't see how this implies that $f$ is monotonic on the entire interval. The same for the interval $(0,1)$.
                          $endgroup$
                          – Servaes
                          Mar 29 at 22:06












                        • $begingroup$
                          @Servaes Thanks! Fixed all.
                          $endgroup$
                          – Yuri Negometyanov
                          Apr 1 at 19:17














                        0












                        0








                        0





                        $begingroup$

                        $color{brown}{textbf{Some forms of the equation.}}$



                        If $underline{xnot=0},$ then unknowns can be swapped. So
                        $$fleft(yf(x)+dfrac xyright) = xyf(x^2+y^2) = fleft(xf(y)+dfrac yxright),tag1$$
                        with the partial cases
                        $$begin{cases}
                        y=1,quad f(f(x)+x) = xf(x^2+1) = fleft(x+dfrac1xright) hspace{226mu}(2.1)\[4pt]
                        y=x,quad f(xf(x)+1) = x^2f(2x^2) hspace{350mu}(2.2)\[4pt]
                        end{cases}$$

                        Denote
                        $$g(x) = xf(x)tag3,$$
                        then from $(2)$ should
                        $$begin{cases}
                        g(1+x^2) = gleft(x+dfrac1xright) hspace{432mu}(4.1)\[4pt]
                        g(g(x)+1) = frac12g(2x^2)(g(x)+1)hspace{370mu}(4.2)\[4pt]
                        end{cases}$$

                        Assume $g(x)$ continuous function.



                        $color{brown}{textbf{Corollaries from the formula (4.1).}}$



                        Using the relationships between the arguments in $(4.1)$ in the form of
                        begin{cases}
                        L_{1,2}=1+x^2=dfrac 12R(Rpmsqrt{R^2-4})\[4pt]
                        R_{1,2}=x+dfrac1x = pmleft(sqrt {L-1}+dfrac1{sqrt{L-1}}right),
                        end{cases}

                        one can present equation $(4.1)$ in the forms of
                        begin{cases}
                        g(x) = gleft(dfrac 12x(xpmsqrt{x^2-4})right)hspace{382mu}(5.1)\[4pt]
                        g(x) = gleft(pmleft(sqrt{x-1} + dfrac1{sqrt{x-1}}right)right).hspace{330mu}(5.2)
                        end{cases}

                        From $(5.2)$ should $g(1)=g(pminfty),$
                        $$g(pminfty)=1.$$
                        Also, formula $(5.2)$ allows to assign to each point of the interval $(1,2)$ the point of the interval $(2,infty)$ with the same value of the function $g.$



                        At the same time, repeated recursive application of formula $(5.1)$ allows to prove that
                        $$g(x)=1quad forall quad xin((-infty,-2]cup[2,infty)).tag6$$



                        $color{brown}{textbf{Corollaries from the formula (4.2).}}$



                        Let us consider such neighbour of the point $x=1,$ where $g(x)>0.$
                        Then the right part of the system $(4.2)$ can be presented in the form of
                        $$g(2x^2)(1+g(x)) = 2.tag7$$
                        Applying $(7)$ for $x$ from $1$ to $+0$ and from $-2$ to $-0,$ easy to see that
                        $g(x)=1.tag8$



                        The value in the singular point $x=0$ can be defined immediately from the equaion $(1)$ and equals to zero.



                        Theerefore, the OP solution
                        $$f(x) =
                        begin{cases}
                        0,quadtext{if}quad x=0\[4pt]
                        dfrac1x,quadtext{otherwize}
                        end{cases}$$

                        is the single non-trivial solution.






                        share|cite|improve this answer











                        $endgroup$



                        $color{brown}{textbf{Some forms of the equation.}}$



                        If $underline{xnot=0},$ then unknowns can be swapped. So
                        $$fleft(yf(x)+dfrac xyright) = xyf(x^2+y^2) = fleft(xf(y)+dfrac yxright),tag1$$
                        with the partial cases
                        $$begin{cases}
                        y=1,quad f(f(x)+x) = xf(x^2+1) = fleft(x+dfrac1xright) hspace{226mu}(2.1)\[4pt]
                        y=x,quad f(xf(x)+1) = x^2f(2x^2) hspace{350mu}(2.2)\[4pt]
                        end{cases}$$

                        Denote
                        $$g(x) = xf(x)tag3,$$
                        then from $(2)$ should
                        $$begin{cases}
                        g(1+x^2) = gleft(x+dfrac1xright) hspace{432mu}(4.1)\[4pt]
                        g(g(x)+1) = frac12g(2x^2)(g(x)+1)hspace{370mu}(4.2)\[4pt]
                        end{cases}$$

                        Assume $g(x)$ continuous function.



                        $color{brown}{textbf{Corollaries from the formula (4.1).}}$



                        Using the relationships between the arguments in $(4.1)$ in the form of
                        begin{cases}
                        L_{1,2}=1+x^2=dfrac 12R(Rpmsqrt{R^2-4})\[4pt]
                        R_{1,2}=x+dfrac1x = pmleft(sqrt {L-1}+dfrac1{sqrt{L-1}}right),
                        end{cases}

                        one can present equation $(4.1)$ in the forms of
                        begin{cases}
                        g(x) = gleft(dfrac 12x(xpmsqrt{x^2-4})right)hspace{382mu}(5.1)\[4pt]
                        g(x) = gleft(pmleft(sqrt{x-1} + dfrac1{sqrt{x-1}}right)right).hspace{330mu}(5.2)
                        end{cases}

                        From $(5.2)$ should $g(1)=g(pminfty),$
                        $$g(pminfty)=1.$$
                        Also, formula $(5.2)$ allows to assign to each point of the interval $(1,2)$ the point of the interval $(2,infty)$ with the same value of the function $g.$



                        At the same time, repeated recursive application of formula $(5.1)$ allows to prove that
                        $$g(x)=1quad forall quad xin((-infty,-2]cup[2,infty)).tag6$$



                        $color{brown}{textbf{Corollaries from the formula (4.2).}}$



                        Let us consider such neighbour of the point $x=1,$ where $g(x)>0.$
                        Then the right part of the system $(4.2)$ can be presented in the form of
                        $$g(2x^2)(1+g(x)) = 2.tag7$$
                        Applying $(7)$ for $x$ from $1$ to $+0$ and from $-2$ to $-0,$ easy to see that
                        $g(x)=1.tag8$



                        The value in the singular point $x=0$ can be defined immediately from the equaion $(1)$ and equals to zero.



                        Theerefore, the OP solution
                        $$f(x) =
                        begin{cases}
                        0,quadtext{if}quad x=0\[4pt]
                        dfrac1x,quadtext{otherwize}
                        end{cases}$$

                        is the single non-trivial solution.







                        share|cite|improve this answer














                        share|cite|improve this answer



                        share|cite|improve this answer








                        edited Apr 1 at 19:09

























                        answered Mar 29 at 21:45









                        Yuri NegometyanovYuri Negometyanov

                        12.5k1729




                        12.5k1729












                        • $begingroup$
                          How do you conclude that $f(-y)=f(y)$ from $(1)$?
                          $endgroup$
                          – Servaes
                          Mar 29 at 21:49










                        • $begingroup$
                          Also, how do you account for the solution $f=0$?
                          $endgroup$
                          – Servaes
                          Mar 29 at 21:54










                        • $begingroup$
                          In fact, if $f$ is monotonic it is injective and then the fact that $$f(yf(x)+tfrac{x}{y})=xyf(x^2+y^2)=(-x)(-y)f((-x)^2+(-y)^2)=f(-yf(-x)+tfrac{-x}{-y}),$$ shows that $f(-x)=-f(x)$ for all $xinBbb{R}$, directly contradicting $(2)$ unless $f=0$.
                          $endgroup$
                          – Servaes
                          Mar 29 at 22:00










                        • $begingroup$
                          Also, I don't see immediately how you prove that $f$ is monotonic on these intervals; your observation partitions $(1,infty)$ into infinite sequences on which $f$ is monotonic, but I don't see how this implies that $f$ is monotonic on the entire interval. The same for the interval $(0,1)$.
                          $endgroup$
                          – Servaes
                          Mar 29 at 22:06












                        • $begingroup$
                          @Servaes Thanks! Fixed all.
                          $endgroup$
                          – Yuri Negometyanov
                          Apr 1 at 19:17


















                        • $begingroup$
                          How do you conclude that $f(-y)=f(y)$ from $(1)$?
                          $endgroup$
                          – Servaes
                          Mar 29 at 21:49










                        • $begingroup$
                          Also, how do you account for the solution $f=0$?
                          $endgroup$
                          – Servaes
                          Mar 29 at 21:54










                        • $begingroup$
                          In fact, if $f$ is monotonic it is injective and then the fact that $$f(yf(x)+tfrac{x}{y})=xyf(x^2+y^2)=(-x)(-y)f((-x)^2+(-y)^2)=f(-yf(-x)+tfrac{-x}{-y}),$$ shows that $f(-x)=-f(x)$ for all $xinBbb{R}$, directly contradicting $(2)$ unless $f=0$.
                          $endgroup$
                          – Servaes
                          Mar 29 at 22:00










                        • $begingroup$
                          Also, I don't see immediately how you prove that $f$ is monotonic on these intervals; your observation partitions $(1,infty)$ into infinite sequences on which $f$ is monotonic, but I don't see how this implies that $f$ is monotonic on the entire interval. The same for the interval $(0,1)$.
                          $endgroup$
                          – Servaes
                          Mar 29 at 22:06












                        • $begingroup$
                          @Servaes Thanks! Fixed all.
                          $endgroup$
                          – Yuri Negometyanov
                          Apr 1 at 19:17
















                        $begingroup$
                        How do you conclude that $f(-y)=f(y)$ from $(1)$?
                        $endgroup$
                        – Servaes
                        Mar 29 at 21:49




                        $begingroup$
                        How do you conclude that $f(-y)=f(y)$ from $(1)$?
                        $endgroup$
                        – Servaes
                        Mar 29 at 21:49












                        $begingroup$
                        Also, how do you account for the solution $f=0$?
                        $endgroup$
                        – Servaes
                        Mar 29 at 21:54




                        $begingroup$
                        Also, how do you account for the solution $f=0$?
                        $endgroup$
                        – Servaes
                        Mar 29 at 21:54












                        $begingroup$
                        In fact, if $f$ is monotonic it is injective and then the fact that $$f(yf(x)+tfrac{x}{y})=xyf(x^2+y^2)=(-x)(-y)f((-x)^2+(-y)^2)=f(-yf(-x)+tfrac{-x}{-y}),$$ shows that $f(-x)=-f(x)$ for all $xinBbb{R}$, directly contradicting $(2)$ unless $f=0$.
                        $endgroup$
                        – Servaes
                        Mar 29 at 22:00




                        $begingroup$
                        In fact, if $f$ is monotonic it is injective and then the fact that $$f(yf(x)+tfrac{x}{y})=xyf(x^2+y^2)=(-x)(-y)f((-x)^2+(-y)^2)=f(-yf(-x)+tfrac{-x}{-y}),$$ shows that $f(-x)=-f(x)$ for all $xinBbb{R}$, directly contradicting $(2)$ unless $f=0$.
                        $endgroup$
                        – Servaes
                        Mar 29 at 22:00












                        $begingroup$
                        Also, I don't see immediately how you prove that $f$ is monotonic on these intervals; your observation partitions $(1,infty)$ into infinite sequences on which $f$ is monotonic, but I don't see how this implies that $f$ is monotonic on the entire interval. The same for the interval $(0,1)$.
                        $endgroup$
                        – Servaes
                        Mar 29 at 22:06






                        $begingroup$
                        Also, I don't see immediately how you prove that $f$ is monotonic on these intervals; your observation partitions $(1,infty)$ into infinite sequences on which $f$ is monotonic, but I don't see how this implies that $f$ is monotonic on the entire interval. The same for the interval $(0,1)$.
                        $endgroup$
                        – Servaes
                        Mar 29 at 22:06














                        $begingroup$
                        @Servaes Thanks! Fixed all.
                        $endgroup$
                        – Yuri Negometyanov
                        Apr 1 at 19:17




                        $begingroup$
                        @Servaes Thanks! Fixed all.
                        $endgroup$
                        – Yuri Negometyanov
                        Apr 1 at 19:17











                        0












                        $begingroup$

                        Let $f:textbf{R}rightarrowtextbf{R}$, such that $f(1)=1$ and
                        $$
                        fleft(yf(x)+frac{x}{y}right)=xyf(x^2+y^2)textrm{, }forall (x,y)intextbf{R}timestextbf{R}^{*}tag 1
                        $$

                        Set $x=0$, $y=1$ in (1), then easily $f(0)=0$ (the case $f(0)neq 0$ is trivial). For $y=1$ in (1) we get
                        $$
                        xf(x^2+1)=f(f(x)+x)textrm{, }xneq 0.tag 2
                        $$

                        Also using the symmetry we get
                        $$
                        fleft(yf(x)+frac{x}{y}right)=fleft(xf(y)+frac{y}{x}right)textrm{, }x,yneq 0
                        $$

                        For $y=1$, we get
                        $$
                        fleft(x+frac{1}{x}right)=f(f(x)+x)textrm{, }xneq 0tag 3
                        $$

                        Assume that $h(x,y)$ is a surface (function) such that
                        $$
                        f(y+h(x,y))=f(x).tag 4
                        $$

                        Note.



                        One choise of $h(x,y)$ is $h(x,y)=-y+x$ but may be there others. For example if $f(x)=x^2-x+1$, then $h(x,y)=x-y$ or $h(x,y)=1-x-y$.



                        We assume here that
                        $$
                        f(x)=hleft(x+frac{1}{x},xright)tag{4.1}
                        $$


                        From (4) with $xrightarrow x+frac{1}{x}$ we get
                        $$
                        fleft(x+frac{1}{x}right)=fleft(y+hleft(x+frac{1}{x},yright)right).
                        $$

                        Hence for $y=x$ in the above identity we get
                        $$
                        fleft(x+frac{1}{x}right)=fleft(x+hleft(x+frac{1}{x},xright)right)=f(x+f(x))
                        $$

                        Hence we can get (3). From (2) and (3) we get also
                        $$
                        xf(x^2+1)=fleft(x+frac{1}{x}right)tag 6
                        $$

                        Setting $xrightarrow x^{-1}$ in (6)
                        $$
                        frac{1}{x}fleft(1+frac{1}{x^2}right)=fleft(x+frac{1}{x}right)=xf(x^2+1).tag 7
                        $$

                        Hence if we set $x^2rightarrow x>0$ in (7), then
                        $$
                        fleft(1+frac{1}{x}right)=xf(x+1)textrm{, for all }x>0.tag 8
                        $$

                        Set also $x=y-1>0$ in (8), then
                        $$
                        fleft(frac{y}{y-1}right)=(y-1)f(y)textrm{, }y>1.
                        $$

                        With $y=1/w>1$ we get
                        $$
                        frac{1}{1-w}fleft(frac{1}{1-w}right)=frac{1}{w}fleft(frac{1}{w}right).
                        $$

                        Hence if $0<w<1$ and $g(w):=frac{1}{w}fleft(frac{1}{w}right)$, then
                        $$
                        f(x)=x^{-1}gleft(x^{-1}right)textrm{, where }x>1textrm{ and }g(1-w)=g(w)textrm{, }0<w<1tag 9
                        $$

                        The solution (9) satisfies (8),(7),(6), but for to holds (3) we get plus a new functional equation for $f(x)$ and this is (4.1).



                        Hence the general solution is
                        $$
                        f(x)=hleft(x+frac{1}{x},xright)tag{11}
                        $$

                        with
                        $$
                        frac{1}{x-1}hleft(x-1+frac{1}{x-1},frac{1}{x-1}right)=frac{1}{x}hleft(x+frac{1}{x},frac{1}{x}right)tag{12}
                        $$

                        and $h(x,y)$ solution of
                        $$
                        f(y+h(x,y))=f(x).tag{13}
                        $$

                        An obvious solution of $(13)$ is $h(x,y)=x-y$, but it may exist and other solutions.






                        share|cite|improve this answer









                        $endgroup$


















                          0












                          $begingroup$

                          Let $f:textbf{R}rightarrowtextbf{R}$, such that $f(1)=1$ and
                          $$
                          fleft(yf(x)+frac{x}{y}right)=xyf(x^2+y^2)textrm{, }forall (x,y)intextbf{R}timestextbf{R}^{*}tag 1
                          $$

                          Set $x=0$, $y=1$ in (1), then easily $f(0)=0$ (the case $f(0)neq 0$ is trivial). For $y=1$ in (1) we get
                          $$
                          xf(x^2+1)=f(f(x)+x)textrm{, }xneq 0.tag 2
                          $$

                          Also using the symmetry we get
                          $$
                          fleft(yf(x)+frac{x}{y}right)=fleft(xf(y)+frac{y}{x}right)textrm{, }x,yneq 0
                          $$

                          For $y=1$, we get
                          $$
                          fleft(x+frac{1}{x}right)=f(f(x)+x)textrm{, }xneq 0tag 3
                          $$

                          Assume that $h(x,y)$ is a surface (function) such that
                          $$
                          f(y+h(x,y))=f(x).tag 4
                          $$

                          Note.



                          One choise of $h(x,y)$ is $h(x,y)=-y+x$ but may be there others. For example if $f(x)=x^2-x+1$, then $h(x,y)=x-y$ or $h(x,y)=1-x-y$.



                          We assume here that
                          $$
                          f(x)=hleft(x+frac{1}{x},xright)tag{4.1}
                          $$


                          From (4) with $xrightarrow x+frac{1}{x}$ we get
                          $$
                          fleft(x+frac{1}{x}right)=fleft(y+hleft(x+frac{1}{x},yright)right).
                          $$

                          Hence for $y=x$ in the above identity we get
                          $$
                          fleft(x+frac{1}{x}right)=fleft(x+hleft(x+frac{1}{x},xright)right)=f(x+f(x))
                          $$

                          Hence we can get (3). From (2) and (3) we get also
                          $$
                          xf(x^2+1)=fleft(x+frac{1}{x}right)tag 6
                          $$

                          Setting $xrightarrow x^{-1}$ in (6)
                          $$
                          frac{1}{x}fleft(1+frac{1}{x^2}right)=fleft(x+frac{1}{x}right)=xf(x^2+1).tag 7
                          $$

                          Hence if we set $x^2rightarrow x>0$ in (7), then
                          $$
                          fleft(1+frac{1}{x}right)=xf(x+1)textrm{, for all }x>0.tag 8
                          $$

                          Set also $x=y-1>0$ in (8), then
                          $$
                          fleft(frac{y}{y-1}right)=(y-1)f(y)textrm{, }y>1.
                          $$

                          With $y=1/w>1$ we get
                          $$
                          frac{1}{1-w}fleft(frac{1}{1-w}right)=frac{1}{w}fleft(frac{1}{w}right).
                          $$

                          Hence if $0<w<1$ and $g(w):=frac{1}{w}fleft(frac{1}{w}right)$, then
                          $$
                          f(x)=x^{-1}gleft(x^{-1}right)textrm{, where }x>1textrm{ and }g(1-w)=g(w)textrm{, }0<w<1tag 9
                          $$

                          The solution (9) satisfies (8),(7),(6), but for to holds (3) we get plus a new functional equation for $f(x)$ and this is (4.1).



                          Hence the general solution is
                          $$
                          f(x)=hleft(x+frac{1}{x},xright)tag{11}
                          $$

                          with
                          $$
                          frac{1}{x-1}hleft(x-1+frac{1}{x-1},frac{1}{x-1}right)=frac{1}{x}hleft(x+frac{1}{x},frac{1}{x}right)tag{12}
                          $$

                          and $h(x,y)$ solution of
                          $$
                          f(y+h(x,y))=f(x).tag{13}
                          $$

                          An obvious solution of $(13)$ is $h(x,y)=x-y$, but it may exist and other solutions.






                          share|cite|improve this answer









                          $endgroup$
















                            0












                            0








                            0





                            $begingroup$

                            Let $f:textbf{R}rightarrowtextbf{R}$, such that $f(1)=1$ and
                            $$
                            fleft(yf(x)+frac{x}{y}right)=xyf(x^2+y^2)textrm{, }forall (x,y)intextbf{R}timestextbf{R}^{*}tag 1
                            $$

                            Set $x=0$, $y=1$ in (1), then easily $f(0)=0$ (the case $f(0)neq 0$ is trivial). For $y=1$ in (1) we get
                            $$
                            xf(x^2+1)=f(f(x)+x)textrm{, }xneq 0.tag 2
                            $$

                            Also using the symmetry we get
                            $$
                            fleft(yf(x)+frac{x}{y}right)=fleft(xf(y)+frac{y}{x}right)textrm{, }x,yneq 0
                            $$

                            For $y=1$, we get
                            $$
                            fleft(x+frac{1}{x}right)=f(f(x)+x)textrm{, }xneq 0tag 3
                            $$

                            Assume that $h(x,y)$ is a surface (function) such that
                            $$
                            f(y+h(x,y))=f(x).tag 4
                            $$

                            Note.



                            One choise of $h(x,y)$ is $h(x,y)=-y+x$ but may be there others. For example if $f(x)=x^2-x+1$, then $h(x,y)=x-y$ or $h(x,y)=1-x-y$.



                            We assume here that
                            $$
                            f(x)=hleft(x+frac{1}{x},xright)tag{4.1}
                            $$


                            From (4) with $xrightarrow x+frac{1}{x}$ we get
                            $$
                            fleft(x+frac{1}{x}right)=fleft(y+hleft(x+frac{1}{x},yright)right).
                            $$

                            Hence for $y=x$ in the above identity we get
                            $$
                            fleft(x+frac{1}{x}right)=fleft(x+hleft(x+frac{1}{x},xright)right)=f(x+f(x))
                            $$

                            Hence we can get (3). From (2) and (3) we get also
                            $$
                            xf(x^2+1)=fleft(x+frac{1}{x}right)tag 6
                            $$

                            Setting $xrightarrow x^{-1}$ in (6)
                            $$
                            frac{1}{x}fleft(1+frac{1}{x^2}right)=fleft(x+frac{1}{x}right)=xf(x^2+1).tag 7
                            $$

                            Hence if we set $x^2rightarrow x>0$ in (7), then
                            $$
                            fleft(1+frac{1}{x}right)=xf(x+1)textrm{, for all }x>0.tag 8
                            $$

                            Set also $x=y-1>0$ in (8), then
                            $$
                            fleft(frac{y}{y-1}right)=(y-1)f(y)textrm{, }y>1.
                            $$

                            With $y=1/w>1$ we get
                            $$
                            frac{1}{1-w}fleft(frac{1}{1-w}right)=frac{1}{w}fleft(frac{1}{w}right).
                            $$

                            Hence if $0<w<1$ and $g(w):=frac{1}{w}fleft(frac{1}{w}right)$, then
                            $$
                            f(x)=x^{-1}gleft(x^{-1}right)textrm{, where }x>1textrm{ and }g(1-w)=g(w)textrm{, }0<w<1tag 9
                            $$

                            The solution (9) satisfies (8),(7),(6), but for to holds (3) we get plus a new functional equation for $f(x)$ and this is (4.1).



                            Hence the general solution is
                            $$
                            f(x)=hleft(x+frac{1}{x},xright)tag{11}
                            $$

                            with
                            $$
                            frac{1}{x-1}hleft(x-1+frac{1}{x-1},frac{1}{x-1}right)=frac{1}{x}hleft(x+frac{1}{x},frac{1}{x}right)tag{12}
                            $$

                            and $h(x,y)$ solution of
                            $$
                            f(y+h(x,y))=f(x).tag{13}
                            $$

                            An obvious solution of $(13)$ is $h(x,y)=x-y$, but it may exist and other solutions.






                            share|cite|improve this answer









                            $endgroup$



                            Let $f:textbf{R}rightarrowtextbf{R}$, such that $f(1)=1$ and
                            $$
                            fleft(yf(x)+frac{x}{y}right)=xyf(x^2+y^2)textrm{, }forall (x,y)intextbf{R}timestextbf{R}^{*}tag 1
                            $$

                            Set $x=0$, $y=1$ in (1), then easily $f(0)=0$ (the case $f(0)neq 0$ is trivial). For $y=1$ in (1) we get
                            $$
                            xf(x^2+1)=f(f(x)+x)textrm{, }xneq 0.tag 2
                            $$

                            Also using the symmetry we get
                            $$
                            fleft(yf(x)+frac{x}{y}right)=fleft(xf(y)+frac{y}{x}right)textrm{, }x,yneq 0
                            $$

                            For $y=1$, we get
                            $$
                            fleft(x+frac{1}{x}right)=f(f(x)+x)textrm{, }xneq 0tag 3
                            $$

                            Assume that $h(x,y)$ is a surface (function) such that
                            $$
                            f(y+h(x,y))=f(x).tag 4
                            $$

                            Note.



                            One choise of $h(x,y)$ is $h(x,y)=-y+x$ but may be there others. For example if $f(x)=x^2-x+1$, then $h(x,y)=x-y$ or $h(x,y)=1-x-y$.



                            We assume here that
                            $$
                            f(x)=hleft(x+frac{1}{x},xright)tag{4.1}
                            $$


                            From (4) with $xrightarrow x+frac{1}{x}$ we get
                            $$
                            fleft(x+frac{1}{x}right)=fleft(y+hleft(x+frac{1}{x},yright)right).
                            $$

                            Hence for $y=x$ in the above identity we get
                            $$
                            fleft(x+frac{1}{x}right)=fleft(x+hleft(x+frac{1}{x},xright)right)=f(x+f(x))
                            $$

                            Hence we can get (3). From (2) and (3) we get also
                            $$
                            xf(x^2+1)=fleft(x+frac{1}{x}right)tag 6
                            $$

                            Setting $xrightarrow x^{-1}$ in (6)
                            $$
                            frac{1}{x}fleft(1+frac{1}{x^2}right)=fleft(x+frac{1}{x}right)=xf(x^2+1).tag 7
                            $$

                            Hence if we set $x^2rightarrow x>0$ in (7), then
                            $$
                            fleft(1+frac{1}{x}right)=xf(x+1)textrm{, for all }x>0.tag 8
                            $$

                            Set also $x=y-1>0$ in (8), then
                            $$
                            fleft(frac{y}{y-1}right)=(y-1)f(y)textrm{, }y>1.
                            $$

                            With $y=1/w>1$ we get
                            $$
                            frac{1}{1-w}fleft(frac{1}{1-w}right)=frac{1}{w}fleft(frac{1}{w}right).
                            $$

                            Hence if $0<w<1$ and $g(w):=frac{1}{w}fleft(frac{1}{w}right)$, then
                            $$
                            f(x)=x^{-1}gleft(x^{-1}right)textrm{, where }x>1textrm{ and }g(1-w)=g(w)textrm{, }0<w<1tag 9
                            $$

                            The solution (9) satisfies (8),(7),(6), but for to holds (3) we get plus a new functional equation for $f(x)$ and this is (4.1).



                            Hence the general solution is
                            $$
                            f(x)=hleft(x+frac{1}{x},xright)tag{11}
                            $$

                            with
                            $$
                            frac{1}{x-1}hleft(x-1+frac{1}{x-1},frac{1}{x-1}right)=frac{1}{x}hleft(x+frac{1}{x},frac{1}{x}right)tag{12}
                            $$

                            and $h(x,y)$ solution of
                            $$
                            f(y+h(x,y))=f(x).tag{13}
                            $$

                            An obvious solution of $(13)$ is $h(x,y)=x-y$, but it may exist and other solutions.







                            share|cite|improve this answer












                            share|cite|improve this answer



                            share|cite|improve this answer










                            answered Apr 2 at 5:41









                            Nikos Bagis Nikos Bagis

                            2,462616




                            2,462616






























                                draft saved

                                draft discarded




















































                                Thanks for contributing an answer to Mathematics Stack Exchange!


                                • Please be sure to answer the question. Provide details and share your research!

                                But avoid



                                • Asking for help, clarification, or responding to other answers.

                                • Making statements based on opinion; back them up with references or personal experience.


                                Use MathJax to format equations. MathJax reference.


                                To learn more, see our tips on writing great answers.




                                draft saved


                                draft discarded














                                StackExchange.ready(
                                function () {
                                StackExchange.openid.initPostLogin('.new-post-login', 'https%3a%2f%2fmath.stackexchange.com%2fquestions%2f3156382%2fsolving-fyfxx-y-xyfx2y2-over-the-reals%23new-answer', 'question_page');
                                }
                                );

                                Post as a guest















                                Required, but never shown





















































                                Required, but never shown














                                Required, but never shown












                                Required, but never shown







                                Required, but never shown

































                                Required, but never shown














                                Required, but never shown












                                Required, but never shown







                                Required, but never shown







                                Popular posts from this blog

                                Magento 2 - Add success message with knockout Planned maintenance scheduled April 23, 2019 at 23:30 UTC (7:30pm US/Eastern) Announcing the arrival of Valued Associate #679: Cesar Manara Unicorn Meta Zoo #1: Why another podcast?Success / Error message on ajax request$.widget is not a function when loading a homepage after add custom jQuery on custom themeHow can bind jQuery to current document in Magento 2 When template load by ajaxRedirect page using plugin in Magento 2Magento 2 - Update quantity and totals of cart page without page reload?Magento 2: Quote data not loaded on knockout checkoutMagento 2 : I need to change add to cart success message after adding product into cart through pluginMagento 2.2.5 How to add additional products to cart from new checkout step?Magento 2 Add error/success message with knockoutCan't validate Post Code on checkout page

                                Fil:Tokke komm.svg

                                Where did Arya get these scars? Unicorn Meta Zoo #1: Why another podcast? Announcing the arrival of Valued Associate #679: Cesar Manara Favourite questions and answers from the 1st quarter of 2019Why did Arya refuse to end it?Has the pronunciation of Arya Stark's name changed?Has Arya forgiven people?Why did Arya Stark lose her vision?Why can Arya still use the faces?Has the Narrow Sea become narrower?Does Arya Stark know how to make poisons outside of the House of Black and White?Why did Nymeria leave Arya?Why did Arya not kill the Lannister soldiers she encountered in the Riverlands?What is the current canonical age of Sansa, Bran and Arya Stark?